Saturday 12 September 2015

calculus - improper (double) integral: $int_0^inftyint_x^inftyfrac{1}{sqrt{t^{3}+1}},mathrm{d}t,mathrm{d}x$

I want to determine if the integral $\,\displaystyle\int_0^\infty\displaystyle\int_x^\infty\frac{1}{\sqrt{t^{3}+1}}\,\mathrm{d}t\,\mathrm{d}x$ converges.



I know that $\displaystyle\int_x^\infty\frac{1}{\sqrt{t^{3}+1}}\,\mathrm{d}t$ converges for all $x \geq 0$ and can show this by the comparison theorem. I just am not sure how to use this fact to justify the convergence or divergence of $\displaystyle\int_0^\infty\displaystyle\int_x^\infty\frac{1}{\sqrt{t^{3}+1}}\,\mathrm{d}t\,\mathrm{d}x$.



Could someone point me in the right direction?



Thanks!

No comments:

Post a Comment

real analysis - How to find $lim_{hrightarrow 0}frac{sin(ha)}{h}$

How to find $\lim_{h\rightarrow 0}\frac{\sin(ha)}{h}$ without lhopital rule? I know when I use lhopital I easy get $$ \lim_{h\rightarrow 0}...